2018-09-03 重点归纳
AMC12是针对高中学生的数学测验,该竞赛开始于2000年,分A赛和B赛,于每年的2月初和2月中举行,学生可任选参加一项即可。其主要目的在于激发学生对数学的兴趣,参予AMC12的学生应该不难发现测验的问题都很具挑战性,但测验的题型都不会超过学生的学习范围。这项测验希望每个考生能从竞赛中享受数学。那么接下来跟随小编来看一下AMC12的官方真题以及官方解答吧:
How many ordered triples  of positive integers satisfy
 of positive integers satisfy  and
 and  ?
?

We prime factorize  and
 and  . The prime factorizations are
. The prime factorizations are  ,
,  and
 and  , respectively. Let
, respectively. Let  ,
,  and
 and  . We know that
. We know that![\[\max(a,d)=3\]](http://latex.artofproblemsolving.com/5/4/1/54144303e6de4ae84935ad1df9a2e569c7041808.png)
![\[\max(b,e)=2\]](http://latex.artofproblemsolving.com/b/7/d/b7de5a5aefbaccf608fb441e29df3ac33370880c.png)
![\[\max(a,g)=3\]](http://latex.artofproblemsolving.com/8/4/f/84f6de12d6a6405690950d990a3222e90f0b561b.png)
![\[\max(b,h)=1\]](http://latex.artofproblemsolving.com/a/8/3/a83271852abe6343879fad35f475114d4bdbb656.png)
![\[\max(c,i)=2\]](http://latex.artofproblemsolving.com/4/0/0/4006861c44f3e57be3b6e3c63512dd130924b51a.png)
![\[\max(d,g)=2\]](http://latex.artofproblemsolving.com/9/4/7/947289a810fdb4c7fec0f66f8d18c74ef47cd61c.png)
 and
and  since
 since  isn't a multiple of 5. Since
 isn't a multiple of 5. Since  we know that
 we know that  . We also know that since
. We also know that since  that
that  . So now some equations have become useless to us...let's take them out.
. So now some equations have become useless to us...let's take them out.![\[\max(b,h)=1\]](http://latex.artofproblemsolving.com/a/8/3/a83271852abe6343879fad35f475114d4bdbb656.png)
![\[\max(d,g)=2\]](http://latex.artofproblemsolving.com/9/4/7/947289a810fdb4c7fec0f66f8d18c74ef47cd61c.png) are the only two important ones left. We do casework on each now. If
are the only two important ones left. We do casework on each now. If  then
 then  or
 or  . Similarly if
. Similarly if  then
 then  . Thus our answer is
. Thus our answer is  .
.
It is well known that if the  and
 and  can be written as
 can be written as  , then the highest power of all prime numbers
, then the highest power of all prime numbers  must divide into either
must divide into either  and/or
 and/or  . Or else a lower
. Or else a lower  is the
 is the  .
.
Start from  :
: so
 so  or
 or  or both. But
 or both. But  because
 because  and
 and  . So
. So  .
.
 can be
 can be  in both cases of
 in both cases of  but NOT
 but NOT  because
 because  and
 and  .
.
So there are six sets of  and we will list all possible values of
 and we will list all possible values of  based on those.
 based on those.
 because
 because  must source all powers of
 must source all powers of  .
.  .
.  because of
 because of  restrictions.
 restrictions.
By different sourcing of powers of  and
 and  ,
,

![\[(8,18):z=300\]](http://latex.artofproblemsolving.com/1/c/f/1cf3a838bf9a20be43aa1ecb285c2dbd9e23bb9b.png)
![\[(8,36):z=75,150,300\]](http://latex.artofproblemsolving.com/6/5/5/655795f2a8a1c29f2e03fc0cc67dbf4aafd51faa.png)
![\[(24,9):z=100,300\]](http://latex.artofproblemsolving.com/9/8/0/980cac2cd20fd620510994b8ffa6d7ea502db80e.png)

![\[(24,36):z=25,50,75,100,150,300\]](http://latex.artofproblemsolving.com/b/b/3/bb3c3eb5b8e65be34878e5c92a5de67b77ce2ee8.png)
 is "enabled" by
 is "enabled" by  sourcing the power of
 sourcing the power of  .
.  is uncovered by
 is uncovered by  sourcing all powers of
 sourcing all powers of  . And
. And  is uncovered by
 is uncovered by  and
 and  both at full power capacity.
 both at full power capacity.
Counting the cases, 
As said in previous solutions, start by factoring  and
 and  . The prime factorizations are as follows:
. The prime factorizations are as follows:![\[72=2^3\cdot 3^2,\]](http://latex.artofproblemsolving.com/3/8/6/386c8a886f135f98dd295c743a0e2895ecd89dfe.png)
![\[600=2^3\cdot 3\cdot 5^2,\]](http://latex.artofproblemsolving.com/1/1/a/11a205724309fd309072aeb375fd46fe53bef494.png)
![\[\text{and } 900=2^2\cdot 3^2\cdot 5^2\]](http://latex.artofproblemsolving.com/c/8/f/c8fc2bd28773d8ca95eef7ff6f1b2646463cab6e.png) To organize
To organize  and their respective LCMs in a simpler way, we can draw a triangle as follows such that
 and their respective LCMs in a simpler way, we can draw a triangle as follows such that  are the vertices and the LCMs are on the edges.
 are the vertices and the LCMs are on the edges.![\[[\textbf{\emph{insert diagram here}}]\]](http://latex.artofproblemsolving.com/5/f/c/5fcb4ca9c271af92d02110fdfb5fb5e5525db8a2.png) Now we can split this triangle into three separate ones for each of the three different prime factors
Now we can split this triangle into three separate ones for each of the three different prime factors  .
. Analyzing for powers of
Analyzing for powers of  , it is quite obvious that
, it is quite obvious that  must have
 must have  as one of its factors since neither
 as one of its factors since neither  can have a power of
 can have a power of  exceeding
exceeding  . Turning towards the vertices
. Turning towards the vertices  , we know at least one of them must have
, we know at least one of them must have  as its factors. Therefore, we have
 as its factors. Therefore, we have  ways for the powers of
 ways for the powers of  for
 for  since the only ones that satisfy the previous conditions are for ordered pairs
 since the only ones that satisfy the previous conditions are for ordered pairs  . Powers of
. Powers of  .
.![\[[\textbf{\emph{insert diagram here for powers of 3}}]\]](http://latex.artofproblemsolving.com/2/3/9/2390e427c9768abbf38911619bc5d07aa6bc04ce.png) Using the same logic as we did for powers of
Using the same logic as we did for powers of  , it becomes quite easy to note that
, it becomes quite easy to note that  must have
 must have  as one of its factors. Moving onto
 as one of its factors. Moving onto  , we can use the same logic to find the only ordered pairs
, we can use the same logic to find the only ordered pairs  that will work are
 that will work are  . Uh oh, where da diagram? The final and last case is the powers of
. Uh oh, where da diagram? The final and last case is the powers of  .
.![\[[\textbf{\emph{insert diagram here for powers of 5}}]\]](http://latex.artofproblemsolving.com/1/d/6/1d6323e527e94fe88eee29b92467e71d5c729910.png) This is actually quite a simple case since we know
This is actually quite a simple case since we know  must have
 must have  as part of its factorization while
 as part of its factorization while  cannot have a factor of
 cannot have a factor of  in their prime factorization.
 in their prime factorization.
Multiplying all the possible arrangements for prime factors  , we get the answer:
, we get the answer:![\[5\cdot3\cdot1=\boxed{\textbf{(A) }15}\]](http://latex.artofproblemsolving.com/7/8/c/78cbf17cc73db8ccdf9a96a62b8ca7bb823cbf0a.png) .
.
Three numbers in the interval ![$\left[0,1\right]$](http://latex.artofproblemsolving.com/a/1/4/a14eeeecde5d3951c357b4a7e280daacdef23ae7.png) are chosen independently and at random. What is the probability that the chosen numbers are the side lengths of a triangle with positive area?
 are chosen independently and at random. What is the probability that the chosen numbers are the side lengths of a triangle with positive area?

WLOG assume  is the largest. Scale the triangle to
 is the largest. Scale the triangle to  or
 or  Then the solution is
 Then the solution is  (Insert graph with square of side length 1 and the line
 (Insert graph with square of side length 1 and the line  that cuts it in half)
 that cuts it in half)
WLOG, let the largest of the three numbers drawn be  . Then the other two numbers are drawn uniformly and independently from the interval
. Then the other two numbers are drawn uniformly and independently from the interval ![$[0,a]$](http://latex.artofproblemsolving.com/6/8/b/68bacdc4b96052586744d16169ccd8c3233ed09f.png) . The probability that their sum is greater than
. The probability that their sum is greater than  is
 is 
When  , consider two cases:
, consider two cases:
1)  , then
, then 
2) , then
, then 
 is the same. Thus the answer is
 is the same. Thus the answer is  .
.
The probability of this occurring is the volume of the corresponding region within a  cube, where each point
 cube, where each point  corresponds to a choice of values for each of
corresponds to a choice of values for each of  and
 and  . The region where, WLOG, side
. The region where, WLOG, side  is too long,
 is too long,  , is a pyramid with a base of area
, is a pyramid with a base of area  and height
 and height  , so its volume is
, so its volume is  . Accounting for the corresponding cases in
. Accounting for the corresponding cases in  and
 and  multiplies our answer by
 multiplies our answer by  , so we have excluded a total volume of
, so we have excluded a total volume of  from the space of possible probabilities. Subtracting this from
 from the space of possible probabilities. Subtracting this from  leaves us with a final answer of
 leaves us with a final answer of  .
.
The probability of this occurring is the volume of the corresponding region within a  cube, where each point
 cube, where each point  corresponds to a choice of values for each of
corresponds to a choice of values for each of  and
 and  . We take a horizontal cross section of the cube, essentially picking a value for z. The area where the triangle inequality will not hold is when
. We take a horizontal cross section of the cube, essentially picking a value for z. The area where the triangle inequality will not hold is when  , which has area
, which has area  or when
 or when  or
 or  , which have an area of
, which have an area of  Integrating this expression from 0 to 1 in the form
 Integrating this expression from 0 to 1 in the form

WLOG assume that  is the largest number and hence the largest side. Then
 is the largest number and hence the largest side. Then  . We can set up a square that is
. We can set up a square that is  by
 by  in the
 in the  plane. We are wanting all the points within this square that satisfy
plane. We are wanting all the points within this square that satisfy  . This happens to be a line dividing the square into 2 equal regions. Thus the answer is
. This happens to be a line dividing the square into 2 equal regions. Thus the answer is  .
.
[][] diagram for this problem goes here (z by z square)
The triangle inequality simplifies to considering only one case:  . Consider the complement (the same statement, except with a less than or equal to). Assume (WLOG)
. Consider the complement (the same statement, except with a less than or equal to). Assume (WLOG)  is the largest, so on average
 is the largest, so on average  (now equal to becomes a degenerate case with probability
 (now equal to becomes a degenerate case with probability  , so we no longer need to consider it). We now want
, so we no longer need to consider it). We now want  , so imagine choosing
, so imagine choosing  at once rather than independently. But we know that
 at once rather than independently. But we know that  is between
 is between  and
 and  . The complement is thus:
. The complement is thus:  . But keep in mind that we choose each
. But keep in mind that we choose each  and
 and  randomly and independently, so if there are
 randomly and independently, so if there are  ways to choose
 ways to choose  together, there are
together, there are  ways to choose them separately, and therefore the complement actually doubles to match each case (a good example of this is to restrict b and c to integers such that if
 ways to choose them separately, and therefore the complement actually doubles to match each case (a good example of this is to restrict b and c to integers such that if  , then we only count this once, but in reality: we have two cases
, then we only count this once, but in reality: we have two cases  , and
, and  ; similar reasoning also generalizes to non-integral values). The complement is then actually
; similar reasoning also generalizes to non-integral values). The complement is then actually  . Therefore, our desired probability is given by
. Therefore, our desired probability is given by 
以上就是小编对AMC12数学竞赛试题及答案的介绍,希望对你有所帮助,更多学习资料请持续关注AMC数学竞赛网!
 
                                            上一篇: AMC 10 |运动的相似三角形
下一篇: AMC考试都适合什么年龄段的学生参加?